Bạn chưa đăng nhập. Vui lòng đăng nhập để hỏi bài

Những câu hỏi liên quan
Ko có tên
Xem chi tiết
Juvia Lockser
Xem chi tiết
MInh NGọc CHu
Xem chi tiết
Trần Minh Hoàng
20 tháng 9 2017 lúc 19:39

Bài 1:

a) 3500 = 3100.5 = (35)100 = 243100

5300 = 5100.3 = (53)100 = 125100

Vì 243100 > 125100 nên 3500 > 5300

b) Không thể biết, nếu n > 100 thì thừa lớn hơn, nếu n < 9 thì thừa bé hơn.

Phạm Hoàng Gia Như
Xem chi tiết
IR IRAN(Islamic Republic...
14 tháng 8 2020 lúc 10:12

https://olm.vn/hoi-dap/detail/260808865927.html

Bài này bạn hỏi rồi nhé ; có nguowifi trà lời rồi

Link đó ;

Khách vãng lai đã xóa
𝓗𝓾𝔂 ♪
Xem chi tiết

# Mik làm ý A trước nhé, mik sợ dài :

- Với n = 1 \(\Rightarrow1=\frac{1.2.3}{6}\)( đúng )

- Giả sử đẳng thức cũng đúng với\(n=k\)hay :

\(1^2+2^2+3^2+...+k^2=\)\(\frac{k\left(k+1\right)\left(2k+1\right)}{6}\)

Ta cần chứng minh nó cũng đúng với\(n=k+1\)hay :

\(1^2+2^2+3^2+...+k^2+\left(k+1\right)^2=\)\(\frac{\left(k+1\right)\left(k+2\right)\left(k+3\right)}{6}\)

Thật vậy, ta có:

\(1^2+2^2+3^2+...+k^2+\left(k+1\right)^2=\)\(\frac{k\left(k+1\right)\left(2k+1\right)}{6}+\left(k+1\right)^2\)

\(\Rightarrow\left(k+1\right)\left(\frac{k\left(2k+1\right)}{6}+k+1\right)=\)\(\left(k+1\right)\left(\frac{2k^2+k+6k+6}{6}\right)\)

\(\Rightarrow\)\(\left(k+1\right)\left(\frac{2k^2+7k+6}{6}\right)=\)\(\frac{\left(k+1\right)\left(k+2\right)\left(2k+3\right)}{6}\)( đpcm )

Khách vãng lai đã xóa

# giờ mik làm ý B nha !

- Với n = 1 \(\Rightarrow\)1 = 1 ( đúng )

Giả sử bài toán đúng với\(n=k\left(n\inℕ^∗\right)\)thì ta có :

1 + 23 + 33 + .... + k3 = \(\left[\frac{n\left(n+1\right)}{2}\right]^2\left(1\right)\)

Ta cần chứng minh đề bài đúng với\(n=k+1\)tức là :

13 + 23 + 33 + ...... + n3 = \(\left[\frac{\left(k+1\right)\left(k+2\right)}{2}\right]^2\left(2\right)\)

Đặt \(B=1^3+2^3+...+\left(k+1\right)^3\)

\(=\left(\frac{k\left(k+1\right)}{2}\right)^2+\left(k+1\right)^3\)theo ( 1 )

\(=\left[\frac{\left(k+1\right)\left(k+2\right)}{2}\right]^2\)theo ( 2 )

\(\Rightarrow\left(1\right),\left(2\right)\)đều đúng

Mà \(\left[\frac{n\left(n+1\right)}{2}\right]^2=\)\(\frac{n^2\left(n+1\right)^2}{4}\)

\(\Rightarrow\)\(1^3+2^3+...+n^3=\)\(\frac{n^2\left(n+1\right)^2}{4}\)( đpcm )

 
Khách vãng lai đã xóa
Công Chúa Họ NGuyễn
Xem chi tiết
ngonhuminh
25 tháng 11 2016 lúc 20:02

a,bc.3=m2,bn

\(\frac{m2,bn}{3}=a,bc\) 

m khong chia duoc cho 3=> m2 chia 3 =a

m khac 2 khac 0=> m=1=> a=4

b chia cho 3 =b => b=9 hoac 0

n chia cho 3=c vay n=3, 9 hoac 6   voi n=3=> c=1 (loai vi co m=1);

 n=6 => c=2 (loai)

vay n=9=> c=3  

KL

a=4; b=0; c=3; m=1; n=9

thu lai

4,03x3=12,09

Pé Kakiku_Oisidu
Xem chi tiết
ST
31 tháng 12 2016 lúc 18:32

n2 + n + 4 chia hết cho n + 1

=> n.n + n + 4 chia hết cho n + 1

=> n(n + 1) + 4 chia hết cho n + 1

Vì n(n + 1) chia hết cho n + 1 nên để n(n + 1) + 4 chia hết cho n + 1 thì 4 chia hết cho n + 1

=> n + 1 thuộc Ư(4) 

=> n + 1 thuộc {1;2;4}

Ta có bảng

n + 1124
n013

Vậy n thuộc {0;1;3}

Pé Kakiku_Oisidu
31 tháng 12 2016 lúc 18:34

Cảm ơn bn ....và bn đồng ý kb vs mik nha......

Cậu Bé Ngu Ngơ
Xem chi tiết
Nguyễn Huyền Phương
13 tháng 10 2016 lúc 14:12

THÔI TỰ ĐI MÀ LÀM NHÌN THẤY LÀ ĐÃ GIẬT MÌNH RỒI DÀI DẰNG DẶC AI MÀ LÀM HẾT ĐƯỢC CÁC BẠN NHỈ !

Băng Dii~
13 tháng 10 2016 lúc 14:35

1 / 

B = 15 + 17 - 16

B = 16

mà 16 không chia hết cho 12 , nên không cần chứng minh cũng ra

2 / 

 a ) N = 1 đó

 b ) N = 1 đó

cách dễ nhất là cứ cho N = 1 , vì bao nhiêu lần 1 thực hiện phép tính chia thì chắng chia hết cho 1

còn lại tương tự nhé !

mình còn làm violympic nữa

Nguyễn Thị Hà anh
1 tháng 5 2017 lúc 21:23

đg là ngu ngơ

T-T        *_*       

Hanh Hong
Xem chi tiết